전자기장에서 전자장과 B 장은 같은 위상입니까?


10

나는 최근 에이 대답을 썼다 .

전파는 전자기 방사선 입니다. 전자기 방사선에는 전기와 자기의 두 가지 구성 요소가 포함됩니다. 위에서 언급했듯이 이러한 구성 요소는 서로를 만듭니다. 빨간색 자기장은 파란색 전기장을 만들어 다음 자기장 등을 만듭니다.

여기에 이미지 설명을 입력하십시오

위키피디아 에서이 다이어그램을 얻었지만 물리 책과 Jim Hawkins WA2WHV 는 같은 다이어그램을 제공합니다.

의견에서 토론은 다음과 같습니다.

Olin Lathrop : 첫 번째 다이어그램이 잘못되었습니다. B와 E 필드는 실제로 그림과 같이 위상이 아닌 서로 90도 위상이 다릅니다. 에너지는 지속적으로 E와 B 필드 사이에서 앞뒤로 움직입니다.

킬란 : 확실합니까? Wikipedia와 나의 물리학 책은 다르게 보입니다. 두 분야는 고정 비율을 가져야한다고 생각합니다. 한 필드는 수평이고 다른 필드는 90도 각도입니다. 다이어그램은 3 차원을 보여주기위한 것입니다.

Olin Lathrop : 흠. 나는 항상 그것들이 직교 안에 있다는 것을 이해했지만 지금은 그것을 볼 시간이 없습니다. 이것은 많은 다른 사람들이 맹목적으로 복제 한 나쁜 다이어그램의 경우 일 수 있습니다. 다이어그램에서 두 필드가 모두 0 일 때 에너지는 어디에 있습니까? 구적법에서, 각 필드 진폭의 제곱의 합은 일정하며, 이는 에너지가 어떻게 지속될 수 있는지에 대한 좋은 설명을 제공합니다. 두 필드 사이에서 앞뒤로 이동하지만 총계는 항상 동일합니다.

나는 Olin의 논리를 따르고 왜 필드가 단계에 있는지 스스로 말할 수 없습니다. 내 질문은 전자기 방사선의 E 및 B 필드가 위상에 있습니까? 이것을 어떻게 이해할 수 있습니까?


"다이어그램에서 두 필드가 모두 0 일 때 에너지는 어디에 있습니까?". 다른 곳. E와 B 필드가 모든 곳에서 순간적으로 0 인 것은 아닙니다.
MSalters

그림에서 자기장은 XY 평면에 있고 전기장은 YZ 평면에 있습니다. (Z가 위로 향한다고 가정하면) 90 도의 위상차가 보이지 않습니까? 내가 틀렸다면 정정하십시오.
매니아

physics.stackexchange.com/questions/461393/…을 보십시오 . E 및 B 파 내에 90 ° 위상의 그림이 있습니다. 이 상황에서 우리는 "결과 E와 B"필드를 위상에 맞지 않을 수 있지만, E와 B를 위상에 둘 다 두 개의 EM 파가 추가 되었기 때문에 반대 방향으로 움직입니다
Alejadro Xalabarder

답변:


8

Maxwell의 방정식에서 완전히 도출 된 내용은 전체 대학 수준의 교과서를 채우며 여기에 들어가기에는 너무 복잡합니다.

그러나 안테나 (선형 도체에 흐르는 전류)의 방사선을 고려할 때 안테나 주위의 E (전기) 및 H (자기) 필드에 몇 가지 다른 구성 요소가 있다는 사실로 귀결됩니다. H 필드의 경우 1 / r 2에 비례하는 한 구성 요소 와 1 / r에 비례하는 다른 구성 요소가 있습니다. E 필드의 경우 1 / r 3 구성 요소, 1 / r 2 구성 요소 및 1 / r 구성 요소의 세 가지가 있습니다.

1 / r 3 항은 용량 성 필드에 저장된 에너지를 나타내는 쌍극자 정전기 장입니다. 유사하게, 1 / r 2 항은 유도장에 저장된 에너지를 나타냅니다. 이것은 안테나 도체의 "자기 인덕턴스"를 나타내며, 전류에 의해 생성 된 자기장이 도체 자체에 "백 EMF"를 유도합니다. 1 / r 항만 실제로 안테나에서 운반되는 에너지를 나타냅니다.

1 / r 3 및 1 / r 2 구성 요소가 우세한 안테나 근처에서 E와 H의 위상 관계는 복잡하며 이러한 필드는 실제로 Olin이 설명하는 방식으로 에너지를 저장하고 에너지를 안테나 자체로 되돌려 보냅니다. .

그러나, "원거리 장"(예를 들어, 안테나로부터 10 개 이상의 파장에서 멀어짐)에서, 장의 1 / r 성분이 우세하여 전파 전자기 평면파를 생성하고, 이들 성분은 실제로 서로 위상이 다르다.


3
나는 E와 H 필드 모두에 대해 1 / r ^ 2에 대해 조금 더 설명하고 싶었습니다.
Andy 일명

주요 질문은 평면 전자기파 (그림에 표시)가 구성 요소 EB의 위상이 있는지 여부입니다. 안테나에서 일어나는 일은 또 다른 주제입니다.
Alejadro Xalabarder

@ AlejadroXalabarder : 실제로는 아닙니다. 어떤 종류의 안테나가 없으면 전자기파를 발사 할 수 없습니다. "평면파"는 안테나의 "원거리 장"에서 일어나는 일을 간략히 보여줍니다. 이것이 Olin의 말과 OP가 읽고있는 내용을 연결하려는 이유입니다.
Dave Tweed

@ 데이브 : 우리는 당신이 알고 있듯이 어디에서나 비행기 파도가 있습니다. EM 생성의 특정 사례 인 무선 안테나에서 발생하는 상황에 대해 이야기하고 있습니다. 그러나이 경우 안테나 근처에서 Maxwell 방정식이 유효 하므로이 특별한 경우 평면 파가 없어도 E와 B가 같은 위상에 있습니다. 실제로 모든 EM 파에 대한 질문은 일반적이며, 가장 일반적이고 실제적인 평면 파를 사용하여 위상 문제를보기가 훨씬 쉽습니다.
Alejadro Xalabarder

5

여유 공간의 임피던스는 일정합니다. 그 값은 E와 H의 비율에 비례합니다.

저항 량이므로 E와 H의 크기가 커지거나 작아 져야합니다.

위키 백과 :- 여기에 이미지 설명을 입력하십시오


1
이것이 핵심입니다 ... 조금만 자세히 설명하겠습니다. EXB 여전히 시간 위상차 경우의 방향을 줄 것이며, 중요한 점은 값이 착체 (실수 및 허수 성분)이 될 것이다 - 즉, "저장"을 갖는다. 순전히 실제 수량은 저항력이 있습니다.
플레이스 홀더

3

혼란은 그것들이 (순시 전기 및 자기 벡터 장) 시간이 아닌 공간에서 90도 떨어져 있다는 사실에서 비롯됩니다. 즉 말하자면:

E × BEB=0 이고 는 전파 방향 (Poynting vector)입니다.E×B


3

z^E=x^E0cos(ωtkz)

×E=tμH
HEHH

기본적으로 질문에 링크 된 것과 같은 다이어그램은 공간의 필드를 시각화하는 데 유용 할 수 있으며 자세히 보면 필드 페이징을 볼 수 있습니다. 방정식을 살펴 보는 것만 큼 공개 할 수 있으며, 수학을 진행하면 Maxwell이 답을 알려줍니다.


2

위키 백과 를 인용하려면 :

필드의 전기 및 자기 부분은 하나의 생성 방식을 지정하는 두 가지 Maxwell 방정식을 충족시키기 위해 고정 된 강도 비율로 서 있습니다. 이 E 및 B 필드도 같은 위상에 있으며 공간의 동일한 지점에서 최대 값과 최소값에 모두 도달합니다 (그림 참조). 일반적인 오해는 전자기 복사의 전자기장과 전자기장이 전자기 복사의 전자기 유도에서 발생하는 것과 같이 전자기 복사에서 전자기 복사의 전자기장과 전자기 복사에서 전자기 유도에서 발생하는 정현파 함수와 같은 위상차를 생성한다는 것입니다. -안테나에 가까이).

전자기파


참고로, 이러한 필드의 방향에 따라 신호의 편광이 결정됩니다. E 및 B 필드의 축을 서로 바꾸면 일부 유형의 안테나는 안테나를 90도 회전시킬 때까지 신호를 수신하지 못합니다. (또는 일부 유형의 선글라스는 신호를 전송하지 못할 수 있습니다)
Brian Drummond

1

전압은 자기장에 의존하지 않고 변화율에 의존합니다. 따라서 유도 전압은 자기장이 0 일 때, 그 파생물이 가장 높을 때 가장 높습니다.

EM 파에서 일정한 에너지를 얻기 위해서는 자기 성분과 전압의 전기 성분이 90도 위상차가되어야합니다. 따라서 전기장이 0 일 때 자기장 의 영향 이 가장 커야합니다. 이것은 필드 자체가 위상에있을 때 발생합니다.


안테나의 유도 전압과 전자기파의 E 필드를 혼동하지 않습니까? 진공 상태에서, B = k̄ / c☓E (파동 방향 k̄)
MSalters

1

그렇습니다. Maxwell 방정식을 사용하여 설명 할 때 "Captainj2001"에 표시된대로 위상 또는 -180 ° 위상입니다.

EBH

EBE×B

ExByzE×B=z^ExByExByEB

당사 사이트를 사용함과 동시에 당사의 쿠키 정책개인정보 보호정책을 읽고 이해하였음을 인정하는 것으로 간주합니다.
Licensed under cc by-sa 3.0 with attribution required.